Sunteți pe pagina 1din 54

UCPB v.

MASAGANA TELAMART
FACTS 1999:
In 1991, UCPB issued 5 fire insurance policies covering Masagana Telamarts
various properties for the period from 22 May 1991 to 22 May 1992.
On March 1992, 2 months before policy expiration, UCPB evaluated the
policies and decided not to renew them upon expiration of their terms on 22
May 1992. UCPB advised Masaganas broker of its intention not to renew the
policies. On April 1992, 1 month before policy expiration], UCPB gave written
notice to Masagana of the non-renewal of the policies. On June 1992 [policy
already expired], Masaganas propertycovered by 3 UCPB-issued policies
was razed by fire.
On 13 July 1992, Masagana presented to UCPBs cashier 5 managers
checks, representing premium for the renewal of the policies for another
year.
It was only on the following day, 14 July 1992, when Masagana filed
with UCPB a formal claim for indemnification of the insured property razed by
fire. On the same day, UCPB returned the 5 managers checks, and rejected
Masaganas claim since the policies had expired and were not renewed, and
the fire occurred on 13 June 1992 (or before tender of premium payment).
Masagana filed a civil complaint for recovery of the face value of
the policies covering the insured property razed by fire. RTC ruled in favor
of Masagana, as it found it to have complied with the obligation to pay the
premium; hence, the replacement-renewal policy of these policies are
effective and binding for another year [22 May 1992 22 May 1993].
CA affirmed RTC, holding that following previous practice, Masagana
was allowed a 60-90 day credit term for the renewal of its policies, and that
the acceptance of the late premium payment suggested that payment could
be made later.
ISSUE & HOLDING
WON the fire insurance policies had expired on 22 May 1992, or had been
extended or renewed by an implied credit arrangement though actual
payment of premium was tendered on a later date after the occurrence of
the risk insured against [fire].
FIRE INSURANCE POLICIES HAD EXPIRED
RATIO
An insurance policy, other than life is not valid and binding until
actual payment of the premium. Any agreement to the contrary is
void.The parties may not agree expressly or impliedly on the extension of
credit or time to pay the premium and consider the policy binding before
actual payment.
The case of Malayan Insurance v. Cruz-Arnaldo cited by the CA is not
applicable. In that case, payment of the premium was made on before the
occurrence of the fire. In the present case, the payment of the premium for

renewal of the policies was tendered a month after the fire occurred.
Masagana did not even give UCPB a notice of loss within a reasonable time
after occurrence of the fire.
CA DECISION REVERSED
2001:
CA disagreed with UCPBs stand that Masaganas tender of payment of
the premiums on 13 July 1992 did not result in the renewal of the policies,
having been made beyond the effective date of renewal as provided under
Policy Condition No. 26:
Renewal Clause. Unless the company at least 45 days in advance of
the end of the policy period mails or delivers to the assured at the address
shown in the policy notice of its intention not to renew the policy or to
condition its renewal upon reduction of limits or elimination of coverages, the
assured shall be entitled to renew the policy upon payment of the premium
due on the effective date of renewal.
The following facts have been established:
1. For years, UCPB had been issuing fire policies to th Masagana, and these
policies were annually renewed.
2. UCPB had been granting Masagana a 60-90-day credit term within which to
pay the premiums on the renewed policies.
3. There was no valid notice of non-renewal of the policies, as there is no proof
that the notice sent by ordinary mail was received by Masagana, and the
copy allegedly sent to Zuellig was ever transmitted to Masagana.
4. The premiums for the policies were paid by Masagana within the 60- 90-day
credit term and were duly accepted and received by UCPBs cashier.
ISSUE & HOLDING
WON IC 77 must be strictly applied to UCPBs advantage despite its practice
of granting a 60- to 90-day credit term for the payment of premiums.
NO. MASAGANA WINS THIS TIME. 1999 DECISION SET ASIDE; CA
DECISION AFFIRMED
SEC. 77. An insurer is entitled to payment of the premium as soon as the
thing insured is exposed to the peril insured against. Notwithstanding any
agreement to the contrary, no policy or contract of insurance issued by an
insurance company is valid and binding unless and until the premium thereof
has been paid, except in the case of a life or an industrial life policy
whenever the grace period provision applies.
SEC. 72. An insurer is entitled to payment of premium as soon as the thing
insured is exposed to the peril insured against, unless there is clear
agreement to grant the insured credit extension of the premium due. No
policy issued by an insurance company is valid and binding unless and until
the premium thereof has been paid. (Underscoring supplied)

IC 77 does not restate the portion of IC 72 expressly permitting


an agreement to extend the periodto pay the premium. However, there
are exceptions to IC 77.
1.
In case of a life or industrial life policywhenever the grace period
provision applies [Sec. 77]
2.
Any acknowledgment of the receipt of premiumis conclusive
evidence of payment [Sec. 78]
3.
If the parties have agreed to the payment ininstallments of the
premium and partial payment has been made at the time of
loss [Makati Tuscany Condominium v. CA]
4.
The insurer may grant credit extensionfor the payment of the
premium [Makati Tuscany Condominium]
5.
Estoppel
IC 77 merely precludes the parties from stipulating that the policy is valid
even if premiums are not paid, but does not expressly prohibit an agreement
granting credit extension, and such an agreement is not contrary to morals,
good customs, public order or public policy. [Makati Tuscany
Condominium v. CA]
ON EXCEPTION #4. If the insurer has granted the insured a credit term for
the payment of the premium and loss occurs before the expiration of the
term, recovery on the policy should be allowed even though the premium is
paid after the loss but within the credit term.
It would be unjust and inequitable if recovery on the policy would not
be permitted against UCPB, which had consistently granted a 60-90-day
credit term for the payment of premiums despite its full awareness of IC 77.
Estoppel bars it from taking refuge under said section, since Masagana relied
in good faith on such practice.
Sec. 8. Unless the policy otherwise provides, where a mortgagor
of property effects insurance in his own name providing that the
loss shall be payable to the mortgagee, or assigns a policy of
insurance to a mortgagee, the insurance is deemed to be upon
the interest of the mortgagor, who does not cease to be a party to
the original contract, and any act of his, prior to the loss, which
would otherwise avoid the insurance, will have the same effect,
although the property is in the hands of the mortgagee, but any
act which, under the contract of insurance, is to be performed by
the mortgagor, may be performed by the mortgagee therein
named, with the same effect as if it had been performed by the
mortgagor.

Sec. 9. If an insurer assents to the transfer of an insurance from a


mortgagor to a mortgagee, and, at the time of his assent,
imposes further obligation on the assignee, making a new
contract with him, the act of the mortgagor cannot affect the
rights of said assignee.
Geagonia v CA G.R. No. 114427 February 6, 1995
Facts:
Geagonia, owner of a store, obtained from Country Bankers fire insurance
policy for P100,000.00. The 1 year policy and covered thestock trading of dry
goods.
The policy noted the requirement that
"3. The insured shall give notice to the Company of any insurance or
insurances already effected, or which may subsequently be effected,
covering any of the property or properties consisting of stocks in trade,
goods in process and/or inventories only hereby insured, and unless notice
be given and the particulars of such insurance or insurances be stated
therein or endorsed in this policy pursuant to Section 50 of the Insurance
Code, by or on behalf of the Company before the occurrence of any loss or
damage, all benefits under this policy shall be deemed forfeited, provided
however, that this condition shall not apply when the total insurance or
insurances in force at the time of the loss or damage is not more than
P200,000.00."
The petitioners stocks were destroyed by fire. He then filed a claim which
was subsequently denied because the petitioners stocks were covered by
two other fire insurance policies for Php 200,000 issued by PFIC. The basis of
the private respondent's denial was the petitioner's alleged violation of
Condition 3 of the policy.
Geagonia then filed a complaint against the private respondent in the
Insurance Commission for the recovery of P100,000.00 under fire insurance
policy and damages. He claimed that he knew the existence of the other two
policies. But, he said that he had no knowledge of the provision in the private
respondent's policy requiring him to inform it of the prior policies and this
requirement was not mentioned to him by the private respondent's agent.
The Insurance Commission found that the petitioner did not violate Condition
3 as he had no knowledge of the existence of the two fire insurance policies
obtained from the PFIC; that it was Cebu Tesing Textiles w/c procured the
PFIC policies w/o informing him or securing his consent; and that Cebu Tesing
Textile, as his creditor, had insurable interest on the stocks.
The Insurance Commission then ordered the respondent company to pay
complainant the sum of P100,000.00 with interest and attorneys fees.

CA reversed the decision of the Insurance Commission because it found that


the petitioner knew of the existence of the two other policies issued by the
PFIC.
Issues:
1. WON the petitioner had not disclosed the two insurance policies when he
obtained the fire insurance and thereby violated Condition 3 of the policy.
2. WON he is prohibited from recovering
Held: Yes. No. Petition Granted
Ratio:
1. The court agreed with the CA that the petitioner knew of the prior policies
issued by the PFIC. His letter of 18 January 1991 to the private respondent
conclusively proves this knowledge. His testimony to the contrary before the
Insurance Commissioner and which the latter relied upon cannot prevail over
a written admission made ante litem motam. It was, indeed, incredible that
he did not know about the prior policies since these policies were not new or
original.
2. Stated differently, provisions, conditions or exceptions in policies which
tend to work a forfeiture of insurance policies should be construed most
strictly against those for whose benefits they are inserted, and most
favorably toward those against whom they are intended to operate.
With these principles in mind, Condition 3 of the subject policy is not totally
free from ambiguity and must be meticulously analyzed. Such analysis leads
us to conclude that (a) the prohibition applies only to double insurance, and
(b) the nullity of the policy shall only be to the extent exceeding P200,000.00
of the total policies obtained.
Furthermore, by stating within Condition 3 itself that such condition shall not
apply if the total insurance in force at the time of loss does not exceed
P200,000.00, the private respondent was amenable to assume a co-insurer's
liability up to a loss not exceeding P200,000.00. What it had in mind was to
discourage over-insurance. Indeed, the rationale behind the incorporation of
"other insurance" clause in fire policies is to prevent over-insurance and thus
avert the perpetration of fraud. When a property owner obtains insurance
policies from two or more insurers in a total amount that exceeds the
property's value, the insured may have an inducement to destroy the
property for the purpose of collecting the insurance. The public as well as the
insurer is interested in preventing a situation in which a fire would be
profitable to the insured.
Tai Tong v Insurance G.R. No. L-55397 February 29, 1988
J. Gancayco

Facts:
Azucena Palomo obtained a loan from Tai Tong Chuache Inc. in the amount of
P100,000.00. To secure the payment of the loan, a mortgage was executed
over the land and the building in favor of Tai Tong Chuache & Co. Arsenio
Chua, representative of Thai Tong Chuache & Co. insured the latter's interest
with Travellers Multi-Indemnity Corporation for P100,000.00 (P70,000.00 for
the building and P30,000.00 for the contents thereof)
Pedro Palomo secured a Fire Insurance Policy covering the building for
P50,000.00 with respondent Zenith Insurance Corporation. On July 16, 1975,
another Fire Insurance was procured from respondent Philippine British
Assurance Company, covering the same building for P50,000.00 and the
contents thereof for P70,000.00.
The building and the contents were totally razed by fire.
Based on the computation of the loss, including the Travellers MultiIndemnity, respondents, Zenith Insurance, Phil. British Assurance and S.S.S.
Accredited Group of Insurers, paid their corresponding shares of the loss.
Complainants were paid the following: P41,546.79 by Philippine British
Assurance Co., P11,877.14 by Zenith Insurance Corporation, and P5,936.57
by S.S.S. Group of Accredited Insurers Demand was made from respondent
Travellers Multi-Indemnity for its share in the loss but the same was refused.
Hence, complainants demanded from the other three (3) respondents the
balance of each share in the loss in the amount of P30,894.31 (P5,732.79Zenith Insurance: P22,294.62, Phil. British: and P2,866.90, SSS Accredited)
but the same was refused, hence, this action.
In their answers, Philippine British Assurance and Zenith Insurance
Corporation denied liability on the ground that the claim of the complainants
had already been waived, extinguished or paid. Both companies set up
counterclaim in the total amount of P 91,546.79.
SSS Accredited Group of Insurers informed the Commission that the claim of
complainants for the balance had been paid in the amount in full.
Travellers Insurance, on its part, admitted the issuance of a Policy and
alleged defenses that Fire Policy, covering the furniture and building of
complainants was secured by a certain Arsenio Chua and that the premium
due on the fire policy was paid by Arsenio Chua.
Tai Tong Chuache & Co. also filed a complaint in intervention claiming the
proceeds of the fire Insurance Policy issued by respondent Travellers MultiIndemnity.
As adverted to above respondent Insurance Commission dismissed spouses
Palomos' complaint on the ground that the insurance policy subject of the
complaint was taken out by Tai Tong Chuache & Company, for its own
interest only as mortgagee of the insured property and thus complainant as
mortgagors of the insured property have no right of action against the

respondent. It likewise dismissed petitioner's complaint in intervention in the


following words:
From the above decision, only intervenor Tai Tong Chuache filed a motion for
reconsideration but it was likewise denied hence, the present petition.
Issue: WON Tai Tong had insurable interest
Held: Yes. Petition granted.
Ratio:
Respondent advanced an affirmative defense of lack of insurable interest on
the part of the petitioner that before the occurrence of the peril insured
against, the Palomos had already paid their credit due the petitioner.
However, they were never able to prove that Tai had a lack of insurable
interest. Hence, the decision must be adverse against them.
However respondent Insurance Commission absolved respondent insurance
company from liability on the basis of the certification issued by the then
Court of First Instance of Davao, Branch II, that in a certain civil action
against the Palomos, Arsenio Lopez Chua stands as the complainant and not
Tai Tong Chuache.
From said evidence respondent commission inferred that the credit extended
by petitioner to the Palomos secured by the insured property must have
been paid. These findings was based upon a mere inference.
The record of the case shows that the petitioner to support its claim for the
insurance proceeds offered as evidence the contract of mortgage which has
not been cancelled nor released. It has been held in a long line of cases that
when the creditor is in possession of the document of credit, he need not
prove non-payment for it is presumed. The validity of the insurance policy
taken by petitioner was not assailed by private respondent. Moreover,
petitioner's claim that the loan extended to the Palomos has not yet been
paid was corroborated by Azucena Palomo who testified that they are still
indebted to herein petitioner.
Public respondent argues however, that if the civil case really stemmed from
the loan granted to Azucena Palomo by petitioner the same should have
been brought by Tai Tong Chuache or by its representative in its own behalf.
From the above premise, respondent concluded that the obligation secured
by the insured property must have been paid. However, it should be borne in
mind that petitioner being a partnership may sue and be sued in its name or
by its duly authorized representative. Petitioner's declaration that Arsenio
Lopez Chua acts as the managing partner of the partnership was
corroborated by respondent insurance company. Thus Chua as the managing
partner of the partnership may execute all acts of administration including
the right to sue debtors of the partnership in case of their failure to pay their
obligations when it became due and demandable. Public respondent's

allegation that the civil case flied by Arsenio Chua was in his capacity as
personal creditor of spouses Palomo has no basis. The policy, then had legal
force and effect.
Sec. 51. A policy of insurance must specify:
(a) The parties between whom the contract is made;
(b) The amount to be insured except in the cases of open or
running policies;
(c) The premium, or if the insurance is of a character where
the exact premium is only determinable upon the termination
of the contract, a statement of the basis and rates upon
which the final premium is to be determined;
(d) The property or life insured;
(e) The interest of the insured in property insured, if he is not
the absolute owner thereof;
(f) The risks insured against; and
(g) The period during which the insurance is to continue.
Sun v CA G.R. No. 89741 March 13, 1991
J. Paras
Facts:
Tan took from Sun Insurance a Php 300,000 policy to cover his electrical
store in Iloilo city. Tans request for an indemnity in 1983 was repeatedly
denied, firstly in 1984. He wrote for a reconsideration in the same year. This
was rejected in 1985, prompting him to file a civil case in the same year. The
insurance company filed a motion to dismiss due to prescription in 1987, but
this was denied. The company went to the court of appeals to petition the
same thing, but this was denied.
Issue:
1. WON the filing of a motion for reconsideration interrupts the twelve
months prescriptive period to contest the denial of the insurance claim.
2. WON the rejection of the claim shall be deemed final only if it contains
words to the effect that denial is final. (ie. the first letter in 1984)
3. When does the cause of action accrue?
Held:
1.No
2.No
3. At the time of the first rejection of the insurance company

Ratio:
1. The policy states in section 27.
Action or suit clause If a claim be made and rejected and an action or suit
be not commenced either in the Insurance Commission or in any court of
competent jurisdiction within twelve (12) months from receipt of notice of
such rejection, or in case of arbitration taking place as provided herein,
within twelve (12) months after due notice of the award made by
thearbitrator or arbitrators or umpire, then the claim shall for all purposes be
deemed to have been abandoned and shall not thereafter be recoverable
hereunder.
Respondent Tan admitted that he received a copy of the letter of rejection
on April 2, 1984. Thus, the 12-month prescriptive period started to run from
the said date of April 2, 1984, under section 27.
2. It was clear in the letter.
Ang v. Fulton Fire Insurance Co.- The condition contained in an insurance
policy that claims must be presented within one year after rejection is not
merely a procedural requirement but an important matter essential to a
prompt settlement of claims against insurance companies as it demands that
insurance suits be brought by the insured while the evidence as to the origin
and cause of destruction have not yet disappeared.
Therefore, there was a necessity of bringing suits against the Insurer within
one year from the rejection of the claim. (1984) The contention of the
respondents that the one-year prescriptive period does not start to run until
the petition for reconsideration had been resolved by the insurer (1985), runs
counter to the doctrine.
The provision in the contract was pursuant to Sec. 63.
A condition, stipulation or agreement in any policy of insurance, limiting the
time for commencing an action thereunder to a period of less than one year
from the time when the cause of action accrues, is void.
3. Eagle star- The right of the insured to the payment of his loss accrues from
the happening of the loss. However, the cause of action in an insurance
contract does not accrue until the insured's claim is finally rejected by the
insurer. This is because before such final rejection there is no real necessity
for bringing suit.
The cause of action, then, started when the insurer denied his claim in the
first instance(1984). This rejection of a petition for reconsideration as
insisted by respondents wasnt the beginning of the cause of action.
Sec. 60. An open policy is one in which the value of the thing
insured is not agreed upon, but is left to be ascertained in case of
loss.

Development Insurance v IAC G.R. No. 71360 July 16, 1986


J. Cruz
Facts:
A fire occurred in the building of Philippine Union. It sued for recovery of
damages from the petitioner on the basis of an insurance contract between
them. The petitioner failed to answer on time despite the numerous
extensions it asked for. It was declared in default by the trial court. A
judgment of default was subsequently rendered on the strength of the
evidence given by the private respondent, which was allowed damages. The
petitioner moved to lift the order of default. Its motion was denied. It went to
the appellate court, which affirmed the decision of the trial court. Hence
this appeal.
Issue: Was Philippine Union required to jointly indemnify the building?
Held: No. Petition dismissed.
Ratio:
The policy insured the private respondent's building against fire for
P2,500,000.00.
The petitioner argued that the respondent must share the difference
between that amount and the face value of the policy and the loss sustained
for 5.8 million under Condition 17 of the policy.
The building was insured at P2,500,000.00 by agreement of the insurer and
the insured.
The agreement is known as an open policy and is subject to the express
condition that:
In the event of loss, whether total or partial, it is understood that
the amount of the loss shall be subject to appraisal and the liability of the
company, if established, shall be limited to the actual loss, subject to
the applicable terms, conditions, warranties and clauses of this Policy, and in
no case shall exceed the amount of the policy.
Section 60 of the Insurance Code defines an open policy is one in which the
value of the thing insured is not agreed upon but is left to be ascertained in
case of loss." This means that the actual loss, as determined, will represent
the total indemnity due the insured from the insurer except only that the
total indemnity shall not exceed the face value of the policy.
The actual loss has been ascertained in this case. Hence, applying the open
policy clause as expressly agreed upon, the private respondent is entitled to
indemnity in the total amount of P508,867.00.
The refusal of its vice-president to receive the private respondent's complaint
was the first indication of the petitioner's intention to prolong this case and

postpone the discharge of its obligation to the private respondent under


this agreement. They still evaded payment for 5 years.
Sec. 61. A valued policy is one which expresses on its face an
agreement that the thing insured shall be valued at a specific sum.
Mrs. Henry Harding vs Commercial Union Assurance Company
38 Phil. 464 Mercantile Law Insurance Law Representation Warranty
In February 1916, Mrs. Harding applied for car insurance for a Studebaker
she received as a gift from her husband. She was assisted by Smith, Bell, and
Co. which was the duly authorized representative (insurance agent) of
Commercial Union Assurance Company in the Philippines. The cars value
was estimated with the help of an experienced mechanic (Mr. Server) of the
Luneta Garage. The car was bought by Mr. Harding for P2,800.00. The
mechanic, considering some repairs done, estimated the value to be at
P3,000.00. This estimated value was the value disclosed by Mrs. Harding to
Smith, Bell, and Co. She also disclosed that the value was an estimate made
by Luneta Garage (which also acts as an agent for Smith, Bell, and Co).
In March 1916, a fire destroyed the Studebaker. Mrs. Harding filed an
insurance claim but Commercial Union denied it as it insisted that the
representations and averments made as to the cost of the car were false;
and that said statement was a warranty. Commercial Union also stated that
the car does not belong to Mrs. Harding because such a gift [from her
husband] is void under the Civil Code.
ISSUE: Whether or not Mrs. Harding is entitled to the insurance claim.
HELD: Yes. Commercial Union is not the proper party to attack the validity of
the gift made by Mr. Harding to his wife.
The statement made by Mrs. Harding as to the cost of the car is not a
warranty. The evidence does not prove that the statement is false. In fact,
the evidence shows that the cost of the car is more than the price of the
insurance. The car was bought for P2,800.00 and then thereafter, Luneta
Garage made some repairs and body paints which amounted to P900.00. Mr.
Server attested that the car is as good as new at the time the insurance was
effected.
Commercial Union, upon the information given by Mrs. Harding, and after an
inspection of the automobile by its examiner, having agreed that it was

worth P3,000, is bound by this valuation in the absence of fraud on the part
of the insured. All statements of value are, of necessity, to a large extent
matters of opinion, and it would be outrageous to hold that the validity of all
valued policies must depend upon the absolute correctness of such
estimated value.

Sec. 79. A person insured is entitled to a return of premium, as


follows:
(a) To the whole premium if no part of his interest in the thing
insured be exposed to any of the perils insured against;
(b) Where the insurance is made for a definite period of time
and the insured surrenders his policy, to such portion of the
premium as corresponds with the unexpired time, at a pro
rata rate, unless a short period rate has been agreed upon
and appears on the face of the policy, after deducting from
the whole premium any claim for loss or damage under the
policy which has previously accrued; Provided, That no holder
of a life insurance policy may avail himself of the privileges of
this paragraph without sufficient cause as otherwise provided
by law.

Plaintiff : LEONA PAULINO Defendant : THE CAPITAL INSURANCE & SURETY


COMPANY, INC.
FACTS:

This is an appeal from the decision of the Court of First Instance of Albay,
dismissing an action for recovery of amount of fire insurance policy.
Paulino was the owner of the JUNIOR CAFE, BAKERY & GROCERY STORE
She accepted a fire insurance policy issued by the defendant and that on
April 30, 1952, the plaintiff wrote the defendant requesting cancellation of
the policy, which the latter received on May 10, 1952

The plaintiff did not return the policy or demanded for the return of the
proportionate premium and neither did the defendant offer to return the
premium
The property covered by the policy was destroyed by fire on August 16,
1952.
The defendant refused to make payment on plaintiff's claim, on the
ground that the policy was cancelled as of May 10, 1952.
Plaintiff contends in this appeal that her letter, dated April 30, 1952, was
a mere request or offer to cancel the policy and did not terminate the
same since it was not accompanied by the surrender of the policy for
cancellation.

ISSUE: W/N Capital Insurance was liable (NO)


RATIO:

This case hinges on the interpretation of paragraph 10 of the policy,


reading:
o This insurance may be terminated at any time at the request of the
Insured, in which case the Company will retain the customary short
period rate for the time the policy has been in force. This insurance
may also at any time be terminated at the option of the Company,
on notice to that effect being given to the Insured, in which case the
Company shall be liable to repay on demand a ratable proportion of
the premium for the expired term from the date of cancelment."
Pursuant to this stipulation, the contract in question could be terminated,
"at any time", upon the unilateral act of either party. Whichever party
exercised the "option", did not need the approval, consent or concurrence
of the other thereto. That consent was given at the time of the making of
the contract. Moreover, pursuant to her letter, plaintiff considered the
contract terminated upon receipt of said letter by the defendant ("desde
el recibo de la presente).
Furthermore, the case of Buckley vs. Citizens Insurance Co. (81 N.E. 165)
relied upon by the plaintiff is not in point. Although the insurance policy
involved in that case contained a clause analogous to the one involved
here, the option was exercised therein, not by the insured, but by the
insurance company, which likewise, requested the return of the policy.
Upon receipt of the communication of the company to this effect, the
insured returned the policy. Subsequently, but before the corresponding
portion of the premium had been refunded to the insured, the property
was destroyed by fire. Upon these facts, the insured was not entitled to

collect the amount of the policy, because the unconditional return thereof
upon request of the company implied "a waiver of his right to treat the
policy as in full force and effect until the company paid or tendered to him
the unearned premium."
Decision affirmed.

Sec. 27. A concealment whether intentional or unintentional


entitles the injured party to rescind a contract of insurance. (As
amended by Batasang Pambansa Blg. 874)
Sec. 29. An intentional and fraudulent omission, on the part of one
insured, to communicate information of matters proving or tending
to prove the falsity of a warranty, entitles the insurer to rescind.

Argente v West Coast G.R. No. L-24899 March 19, 1928


J. Malcolm

Facts:
Bernardo Argente signed an application for joint insurance with his wife in
the sum of P2,000. The wife, Vicenta de Ocampo, signed for the same. All the
information contained in the applications was furnished the agent by
Bernardo Argente.
Argente was examined by Dr. Sta. Ana, a medical examiner for the West
Coast. The result was recorded in the Medical Examiner's Report, and with
the exception of the signature of Bernardo Argente, was in the hand-writing
of Doctor Sta. Ana. But the information or answers to the questions
contained on the face of the Medical Examiner's Report were furnished the
doctor by Argente.
Vicenta de Ocampo, wife of the plaintiff, was examined at her residence by
the same doctor.
The spouses submitted to West Coast Life an amended application,
increasing the amount to P15,000, and asked that the policy be dated May
15, 1925. The amended application was accompanied by the documents
entitled "Short Form Medical Report." In both of these documents appear
certain questions and answers.
A temporary policy for P15,000 was issued to Bernardo Argente and his wife
as of May 15, but it was not delivered until the first quarterly premium on the

policy was paid. More than thirty days had elapsed since the applicants were
examined. Each of them was required to file a certificate of health before the
policy was delivered.
Vicenta de Ocampo died of cerebral apoplexy. Argente presented a claim in
due form to the West Coast Life Insurance Co. for the payment of the sum of
P15,000. It was apparently disclosed that the answers given by the insured in
their medical examinations with regard to their health were untrue. West
Coastrefused to pay the claim and wrote Argente to the effect that the claim
was rejected due to fraud.
The trial court held the policy null and void, hence this appeal.
Issue: WON Argente and Ocampo were guilty of concealment and thereby
misled the insurer into accepting the risk?
Held: Yes. Petition dismissed.

Ratio:
Vicenta de Ocampo, in response to the question asked by the medical
examiner, answered no to "Have you ever consulted a physician for or have
you ever suffered from any ailment or disease of the brain or nervous
system?" She also answered none as to the question whether she
consumed alcohol of not.
To the question, "What physician or physicians, if any, not named above,
have you consulted or been treated by, within the last five years and for
what illness or ailment?" she answered "None."
But the facts show that she was taken to San Lazaro Hospital, her case was
diagnosed by the admitting physician as "alcoholism, moreover, she was
diagnosed with "phycho-neurosis."
Section 25 of the Insurance Code defined concealment as "a neglect to
communicate that which a party knows and ought to communicate."
The court held that the alleged concealment was not immaterial and
insufficient to avoid the policy. In an action on a life insurance policy where
the evidence conclusively shows that the answers to questions concerning
diseases were untrue, the truth of falsity of the answers become the
determining factor. If the true facts been disclosed by the assured, the
insurance would never have been granted.
Concealment must, in the absence of inquiries, be not only material, but
fraudulent, or the fact must have been intentionally withheld. If no inquiries

are made and no fraud or design to conceal enters into the concealment the
contract is not avoided.
The assurer is entitled to know every material fact of which the assured has
exclusive or peculiar knowledge, as well as all material facts which directly
tend to increase the hazard or risk which are known by the assured, or which
ought to be or are presumed to be known by him. And a concealment of such
facts vitiates the policy.
If the assured has exclusive knowledge of material facts, he should fully and
fairly disclose the same, whether he believes them material or not. The
determination of the point whether there has or has not been a material
concealment must rest largely in all cases upon the exact terms of the
contract.

Great Pacific Life v CA and Teodoro Cortez, G.R. No. L-57308, April
23, 1990
Issue: Whether the insured may claim the refund of the premium he has
paid
Ruling: The insured is entitled to the refund of the premium paid including
damages.
Basis: When the petitioner advised private respondent on June 1, 1973, four
months after he had paid the first premium, that his policy had never been in
force, and that he must pay another premium and undergo another medical
examination to make the policy effective, the petitioner committed a serious
breach of the contract of insurance.
Petitioner should have informed Cortez of the deadline for paying the first
premium before or at least upon delivery of the policy to him, so he could
have complied with what was needful and would not have been misled into
believing that his life and his family were protected by the policy, when
actually they were not. And, if the premium paid by Cortez was unacceptable
for being late, it was the company's duty to return it. By accepting his
premiums without giving him the corresponding protection, the company
acted in bad faith.
Sections 79, 81 and 82 of P.D. 612 of the Insurance Code of 1978 provide
when the insured is entitled to the return of premium paid.

SECTION 79. A person insured is entitled to a return of premium,


as follows:
(a) To the whole premium, if no part of his interest in the thing
insured be exposed to any of the perils insured against.
(b) Where the insure is made for a definite period of time and the
insured surrenders his policy, to such portion of the premium as
corresponds with the unexpired time, at a pro rata rate, unless a
short period rate has been agreed upon and appears on the face
of the policy, after deducting from the whole premium any claim
for loss or damage under the policy which has previously
accrued: Provided,That no holder of a life insurance policy may
avail himself of the privileges of this paragraph without sufficient
causes as otherwise provided by law.
SECTION 81. A person insured is entitled to a return of the
premium when the contract is voidable on account of the fraud
or misrepresentation of the insurer or of his agent or on account
of facts the existence of which the insured was ignorant without
his fault; or when, by any default of the insured other than actual
fraud, the insurer never incurred any liability under the policy.
SECTION 82. In case of an over-insurance by several insurers, the
insured is entitled to a ratable return of the premium,
proportioned to the amount by which the aggregate sum insured
in all the policies exceeds the insurable value of the thing at risk.
Since the policy was in fact inoperative or ineffectual from the beginning, the
company was never at risk, hence, it is not entitled to keep the premium.
The award of moral damages to Cortez was proper for there can hardly be
any doubt that he must have suffered moral shock, serious anxiety and
wounded feelings upon being informed by the petitioner six (6) months after
it issued the policy to him and four (4) months after receiving the full
premium, that his policy was in fact worthless for it never took effect, hence,
he and his family never received the protection that he paid for.

Title 4
CONCEALMENT

Sec. 26. A neglect to communicate that which a party knows and


ought to communicate, is called a concealment.
Sec. 27. A concealment whether intentional or unintentional
entitles the injured party to rescind a contract of insurance. (As
amended by Batasang Pambansa Blg. 874)
Sec. 28. Each party to a contract of insurance must communicated
to the other, in good faith, all facts within his knowledge which
are material to the contract and as to which he makes no
warranty, and which the other has not the means of ascertaining.
Sec. 29. An intentional and fraudulent omission, on the part of
one insured, to communicate information of matters proving or
tending to prove the falsity of a warranty, entitles the insurer to
rescind.
Sec. 30. Neither party to a contract of insurance is bound to
communicate information of the matters following, except in
answer to the inquiries of the other:
(a) Those which the other knows;
(b) Those which, in the exercise of ordinary care, the other
ought to know, and of which the former has no reason to
suppose him ignorant;
(c) Those of which the other waives communication;
(d) Those which prove or tend to prove the existence of a risk
excluded by a warranty, and which are not otherwise
material; and
(e) Those which relate to a risk excepted from the policy and
which are not otherwise material.
Sec. 31. Materiality is to be determined not by the event, but
solely by the probable and reasonable influence of the facts upon
the party to whom the communication is due, in forming his
estimate of the disadvantages of the proposed contract, or in
making his inquiries.
Sec. 32. Each party to a contract of insurance is bound to know all
the general causes which are open to his inquiry, equally with
that of the other, and which may affect the political or material
perils contemplated; and all general usages of trade.
Sec. 33. The right to information of material facts may be waived,
either by the terms of the insurance or by neglect to make inquiry
as to such facts, where they are distinctly implied in other facts of
which information is communicated.
Sec. 34. Information of the nature or amount of the interest of
one insured need not be communicated unless in answer to an
inquiry, except as prescribed by section fifty-one.

Sec. 35. Neither party to a contract of insurance is bound to


communicate, even upon inquiry, information of his own judgment
upon the matters in question.

Ng v Asian Crusader G.R. No. L-30685 May 30, 1983


J. Escolin:
Facts:
Kwong Nam applied for a 20-year endowment insurance on his life for the
sum of P20,000.00, with his wife, appellee Ng Gan Zee as beneficiary. On the
same date, Asian Crusader, upon receipt of the required premium from the
insured, approved the application and issued the corresponding policy.
Kwong Nam died of cancer of the liver with metastasis. All premiums had
been paid at the time of his death.
Ng Gan Zee presented a claim for payment of the face value of the policy. On
the same date, she submitted the required proof of death of the
insured. Appellant denied the claim on the ground that the answers given by
the insured to the questions in his application for life insurance were untrue.
Appellee brought the matter to the attention of the Insurance Commissioner.
The latter, after conducting an investigation, wrote the appellant that he had
found no material concealment on the part of the insured and that, therefore,
appellee should be paid the full face value of the policy. The company
refused to settle its obligation.
Appellant alleged that the insured was guilty of misrepresentation when
he answered "No" to the following question appearing in the application for
life insuranceHas any life insurance company ever refused your application for insurance
or for reinstatement of a lapsed policy or offered you a policy different from
that applied for? If, so, name company and date.
The lower court ruled against the company on lack of evidence.
Appellant further maintains that when the insured was examined in
connection with his application for life insurance, he gave the appellant's
medical examiner false and misleading information as to his ailment and
previous operation. The company contended that he was operated on for
peptic ulcer 2 years before the policy was applied for and that he never
disclosed such an operation.

Issue: WON Asian Crusader was deceived into entering the contract or in
accepting the risk at the rate of premium agreedupon because of insured's
representation?

Held: No. Petition dismissed.

Ratio:
Section 27 of the Insurance Law:
Sec. 27. Such party a contract of insurance must communicate to the other,
in good faith, all facts within his knowledge which are material to the
contract, and which the other has not the means of ascertaining, and as to
which he makes no warranty.
"Concealment exists where the assured had knowledge of a fact material to
the risk, and honesty, good faith, and fair dealing requires that he should
communicate it to the assurer, but he designedly and intentionally withholds
the same."
It has also been held "that the concealment must, in the absence of
inquiries, be not only material, but fraudulent, or the fact must have been
intentionally withheld."
Fraudulent intent on the part of the insured must be established to entitle
the insurer to rescind the contract. And as correctly observed by the lower
court, "misrepresentation as a defense of the insurer to avoid liability is an
'affirmative' defense. The duty to establish such a defense by satisfactory
and convincing evidence rests upon the defendant. The evidence before the
Court does not clearly and satisfactorily establish that defense."
It bears emphasis that Kwong Nam had informed the appellant's medical
examiner of the tumor. His statement that said tumor was "associated with
ulcer of the stomach" should be construed as an expression made in good
faith of his belief as to the nature of his ailment and operation.
While the information communicated was imperfect, the same was sufficient
to have induced appellant to make further inquiries about the ailment and
operation of the insured.
Section 32 of Insurance Law:
Section 32. The right to information of material facts maybe waived either by
the terms of insurance or by neglect to make inquiries as to such facts where
they are distinctly implied in other facts of which information is
communicated.

Where a question appears to be not answered at all or to be


imperfectly answered, and the insurers issue a policy without any further
inquiry, they waive the imperfection of the answer and render the omission
to answer more fully immaterial.
The company or its medical examiner did not make any further inquiries on
such matters from the hospital before acting on the application for insurance.
The fact of the matter is that the defendant was too eager to accept the
application and receive the insured's premium. It would be inequitable now
to allow the defendant to avoid liability under the circumstances."
Vda Canilang v CA G.R. No. 92492 June 17, 1993
J. Feliciano
Facts:
Canilang was found to have suffered from sinus tachycardia then bronchitis
after a check-up from his doctor. The next day, he applied for a "nonmedical" insurance policy with respondent Grepalife naming his wife, Thelma
Canilang, as his beneficiary. This was to the value of P19,700.
He died of "congestive heart failure," "anemia," and "chronic anemia." The
widow filed a claim with Great Pacific which the insurer denied on the ground
that the insured had concealed material information from it.
Petitioner then filed a complaint against Great Pacific for recovery of the
insurance proceeds. Petitioner testified that she was not aware of any serious
illness suffered by her late husband and her husband had died because of a
kidney disorder. The doctor who gave the check up stated that he treated the
deceased for sinus tachycardia and "acute bronchitis."
Great Pacific presented a physician who testified that the deceased's
insurance application had been approved on the basis of his medical
declaration. She explained that as a rule, medical examinations are required
only in cases where theapplicant has indicated in his application for
insurance coverage that he has previously undergone medical consultation
and hospitalization.
The Insurance Commissioner ordered Great Pacific to pay P19,700 plus legal
interest and P2,000.00 as attorney's fees. On appeal by Great Pacific, the
Court of Appeals reversed. It found that the failure of Jaime Canilang to
disclose previous medical consultation and treatment constituted material
information which should have been communicated to Great Pacific to enable
the latter to make proper inquiries.
Hence this petition by the widow.

Issue: Won Canilang was guilty of misrepresentation


Held: Yes. Petition denied.
Ratio: There was a right of the insurance company to rescind the contract if it
was proven that the insured committed fraud in not affirming that he was
treated for heart condition and other ailments stipulated.
Apart from certifying that he didnt suffer from such a condition, Canilang
also failed to disclose in the that he had twice consulted a doctor who had
found him to be suffering from "sinus tachycardia" and "acute bronchitis."
Under the Insurance Code:
Sec. 26. A neglect to communicate that which a party knows and ought to
communicate, is called a concealment.
Sec. 28. Each party to a contract of insurance must communicate to the
other, in good faith, all factors within his knowledge which are material to the
contract and as to which he makes no warranty, and which the other has not
the means of ascertaining.
The information concealed must be information which the concealing party
knew and should have communicated. The test of materiality of such
information is contained in Section 31:
Sec. 31. Materiality is to be determined not by the event, but solely by the
probable and reasonable influence of the facts upon the party to whom the
communication is due, in forming his estimate of the disadvantages of the
proposed contract, or in making his inquiries.
The information which Jaime Canilang failed to disclose was material to
the ability of Great Pacific to estimate the probable risk he presented as a
subject of life insurance. Had he disclosed his visits to his doctor, the
diagnosis made and medicines prescribed by such doctor, in the insurance
application, it may be reasonably assumed that Great Pacific would have
made further inquiries and would have probably refused to issue a nonmedical insurance policy.
Materiality relates rather to the "probable and reasonable influence of the
facts" upon the party to whom the communication should have been made,
in assessing the risk involved in making or omitting to make further inquiries
and in accepting the application for insurance; that "probable and reasonable
influence of the facts" concealed must, of course, be determined objectively,
by the judge ultimately.
The Insurance Commissioner had also ruled that the failure of Great Pacific to
convey certain information to the insurer was not "intentional" in nature, for

the reason that Canilang believed that he was


minor ailment like a common cold. Section 27 stated that:

suffering

from

Sec. 27. A concealment whether intentional or unintentional entitles the


injured party to rescind a contract of insurance.
The failure to communicate must have been intentional rather than
inadvertent. Canilang could not have been unaware that his heart beat would
at times rise to high and alarming levels and that he had consulted a doctor
twice in the two (2) months before applying for non-medical insurance.
Indeed, the last medical consultation took place just the day before the
insurance application was filed. In all probability, Jaime Canilang went to visit
his doctor precisely because of the ailment.
Canilang's failure to set out answers to some of the questions in the
insurance application constituted concealment.

Sun Life v. CA - Concealment in Insurance


245 SCRA 268 (1995)
Facts:
> On April 15, 1986, Bacani procured a life insurance contract for himself
from Sun Life. He was issued a life insurance policy with double indemnity in
case of accidental death. The designated beneficiary was his mother,
Bernarda.
> On June 26, 1987, the insured died in a plane crash. Bernarda Bacani filed
a claim with Sun Life, seeking the benefits of the insurance. Sun Life
conducted an investigation and its findings prompted it to reject the claim.
> Sun Life discovered that 2 weeks prior to his application, Bacani was
examined and confined at the Lung Center of the Philippines, where he was
diagnosed for renal failure. During his confinement, the deceased was
subjected to urinalysis, ultra-sonography and hematology tests. He did not
reveal such fact in his application.
> In its letter, Sun Life informed Berarda, that the insured did not disclosed
material facts relevant to the issuance of the policy, thus rendering the
contract of insurance voidable. A check representing the total premiums paid
in the amount of P10,172.00 was attached to said letter.

> Bernarda and her husband, filed an action for specific performance
against Sun Life. RTC ruled for Bernarda holding that the facts concealed by
the insured were made in good faith and under the belief that they need not
be disclosed. Moreover, it held that the health history of the insured was
immaterial since the insurance policy was "non-medical." CA affirmed.
Issue:
Whether or not the beneficiary can claim despite the concealment.
Held:
NOPE.
Section 26 of the Insurance Code is explicit in requiring a party to a contract
of insurance to communicate to the other, in good faith, all facts within his
knowledge which are material to the contract and as to which he makes no
warranty, and which the other has no means of ascertaining.
Materiality is to be determined not by the event, but solely by the probable
and reasonable influence of the facts upon the party to whom
communication is due, in forming his estimate of the disadvantages of the
proposed contract or in making his inquiries (The Insurance Code, Sec 31)
The terms of the contract are clear. The insured is specifically required to
disclose to the insurer matters relating to his health. The information which
the insured failed to disclose were material and relevant to the approval and
the issuance of the insurance policy. The matters concealed would have
definitely affected petitioner's action on his application, either by approving
it with the corresponding adjustment for a higher premium or rejecting the
same. Moreover, a disclosure may have warranted a medical examination of
the insured by petitioner in order for it to reasonably assess the risk involved
in accepting the application.
Thus, "good faith" is no defense in concealment.+ The insured's failure to
disclose the fact that he was hospitalized for two weeks prior to filing his
application for insurance, raises grave doubts about his bonafides. It appears
that such concealment was deliberate on his part.
Great Pacific v CA G.R. No. L-31845 April 30, 1979
J. De Castro

Facts:
Ngo Hing filed an application with the Great Pacific for a twenty-year
endowment policy in the amount of P50,000.00 on the life of his one-year old
daughter Helen. He supplied the essential data which petitioner Mondragon,
the Branch Manager, wrote on the form. The latter paid the annual premium
the sum of P1,077.75 going over to the Company, but he retained the
amount of P1,317.00 as his commission for being a duly authorized agent of
Pacific Life.
Upon the payment of the insurance premium, the binding deposit receipt
was issued Ngo Hing. Likewise, petitioner Mondragon handwrote at the
bottom of the back page of the application form his strong recommendation
for the approval of the insurance application. Then Mondragon received a
letter from Pacific Life disapproving the insurance application. The letter
stated that the said life insurance application for 20-year endowment plan is
not available for minors below seven years old, but Pacific Life can consider
the same under the Juvenile Triple Action Plan, and advised that if the offer
is acceptable, the Juvenile Non-Medical Declaration be sent to the company.
The non-acceptance of the insurance plan by Pacific Life was allegedly not
communicated by petitioner Mondragon to private respondent Ngo Hing.
Instead, on May 6, 1957, Mondragon wrote back Pacific Life again strongly
recommending the approval of the 20-year endowment insurance plan to
children, pointing out that since the customers were asking for such
coverage.
Helen Go died of influenza. Ngo Hing sought the payment of the proceeds of
the insurance, but having failed in his effort, he filed the action for the
recovery before the Court of First Instance of Cebu, which ruled against him.

Issues:
1. Whether the binding deposit receipt constituted a temporary contract of
the life insurance in question
2. Whether Ngo Hing concealed the state of health and physical condition of
Helen Go, which rendered void the policy

Held: No. Yes. Petition dismissed.


Ratio:
The receipt was intended to be merely a provisional insurance contract. Its
perfection was subject to compliance of the following conditions: (1) that the

company shall be satisfied that the applicant was insurable on


standard rates; (2) that if the company does not accept the application and
offers to issue a policy for a different plan, the insurance contract shall not
be binding until the applicant accepts the policy offered; otherwise, the
deposit shall be refunded; and (3) that if the company disapproves the
application, the insurance applied for shall not be in force at any time, and
the premium paid shall be returned to the applicant.
The receipt is merely an acknowledgment that the latter's branch office had
received from the applicant the insurance premium and had accepted the
application subject for processing by the insurance company. There was still
approval or rejection the same on the basis of whether or not the applicant is
"insurable on standard rates." Since Pacific Life disapproved the insurance
application of respondent Ngo Hing, the binding deposit receipt in question
had never become in force at any time. The binding deposit receipt is
conditional and does not insure outright. This was held in Lim v Sun.
The deposit paid by private respondent shall have to be refunded by Pacific
Life.
2. Ngo Hing had deliberately concealed the state of health of his daughter
Helen Go. When he supplied data, he was fully aware that his one-year old
daughter is typically a mongoloid child. He withheld the fact material to the
risk insured.
The contract of insurance is one of perfect good faith uberrima fides
meaning good faith, absolute and perfect candor or openness and honesty;
the absence of any concealment or demotion, however slight.
The concealment entitles the insurer to rescind the contract of insurance.
Sec. 48. Whenever a right to rescind a contract of insurance is given
to the insurer by any provision of this chapter, such right must be
exercised previous to the commencement of an action on the
contract.
Sec. 227. In the case of individual life or endowment insurance,
the policy shall contain in substance the following conditions:
(a) A provision that the policyholder is entitled to a grace
period either of thirty days or of one month within which the
payment of any premium after the first may be made, subject
at the option of the insurer to an interest charge not in
excess of six per centum per annum for the number of days of
grace elapsing before the payment of the premium, during
which period of grace the policy shall continue in full force,
but in case the policy becomes a claim during the said period
of grace before the overdue premium is paid, the amount of

such premium with interest may de deducted from the


amount payable under the policy in settlement;
(b) A provision that the policy shall be incontestable after it
shall have been in force during the lifetime of the insured for
a period of two years from its date of issue as shown in the
policy, or date of approval of last reinstatement, except for
non-payment of premium and except for violation of the
conditions of the policy relating to military or naval service in
time of war;
(c) A provision that the policy shall constitute the entire
contract between the parties, but if the company desires to
make the application a part of the contract it may do so
provided a copy of such application shall be indorsed upon or
attached to the policy when issued, and in such case the
policy shall contain a provision that the policy and the
application therefore shall constitute the entire contract
between the parties;
(d) A provision that if the age of the insured is considered in
determining the premium and the benefits accruing under the
policy, and the age of the insured has been misstated, the
amount payable under the policy shall be such as the
premium would have purchased at the correct age;
(e) If the policy is participating, a provision that the company
shall periodically ascertain and apportion any divisible
surplus accruing on the policy under conditions specified
therein;
(f) A provision specifying the options to which the
policyholder is entitled to in the event of default in a premium
payment after three full annual premiums shall have been
paid. Such option shall consist of:
(1) A cash surrender value payable upon surrender of
the policy which shall not be less than the reserve on
the policy, the basis of which shall be indicated, for the
then current policy year and any dividend additions
thereto, reduced by a surrender charge which shall not
be more than one-fifth of the entire reserve or two and
one-half per centum of the amount insured and any
dividend additions thereto;
(2) One or more paid-up benefits on a plan or plans
specified in the policy of such value as may be
purchased by the cash surrender value;
(g) A provision that at anytime after a cash surrender value is
available under the policy and while the policy is in force, the

company will advance, on proper assignment or pledge of the


policy and on sole security thereof, a sum equal to, or at the
option of the owner of the policy, less than the cash surrender
value on the policy, at a specified rate of interest, not more
than the maximum allowed by law, to be determined by the
company from time to time, but not more often than once a
year, subject to the approval of the Commissioner; and that
the company will deduct from such loan value any existing
indebtedness on the policy and any unpaid balance of the
premium for the current policy year, and may collect interest
in advance on the loan to the end of the current policy year,
which provision may further provide that such loan may be
deferred for not exceeding six months after the application
therefore is made;
(h) A table showing in figures cash surrender values and paidup options available under the policy each year upon default
in premium payments, during at least twenty years of the
policy beginning with the year in which the values and
options first become available, together with a provision that
in the event of the failure of the policyholder to elect one of
the said options within the time specified in the policy, one of
said options shall automatically take effect and no
policyholder shall ever forfeit his right to same by reason of
his failure to so elect;
(i) In case the proceeds of a policy are payable in installments
or as an annuity, a table showing the minimum amounts of
the installments or annuity payments;
(j) A provision that the policyholder shall be entitled to have
the policy reinstated at any time within three years from the
date of default of premium payment unless the cash
surrender value has been duly paid, or the extension period
has expired, upon production of evidence of insurability
satisfactory to the company and upon payment of all overdue
premiums and any indebtedness to the company upon said
policy, with interest rate not exceeding that which would have
been applicable to said premiums and indebtedness in the
policy years prior to reinstatement.
Any of the foregoing provisions or portions thereof not applicable
to single premium or term policies shall to that extent not be
incorporated therein; and any such policy may be issued and
delivered in the Philippines which in the opinion of the
Commissioner contains provisions on any one or more of the
foregoing requirements more favorable to the policyholder than
hereinbefore required.

This section shall not apply to policies of group life or industrial


life insurance.
Tan v CA G.R. No. 48049 June 29, 1989
J. Gutierrez Jr.
Facts:
Tan Lee Siong, father of the petitioners, applied for life insurance in the
amount of P 80,000.00 with Philamlife. It was approved. Tan Lee Siong died
of hepatoma. Petitioners then filed a claim for the proceeds. The company
denied petitioners' claim and rescinded the policy by reason of
the alleged misrepresentation and concealment of material facts. The
premiums paid on the policy were refunded. The petitioners filed a complaint
in the Insurance Commission. The latter dismissed the complaint.
The Court of Appeals dismissed ' the petitioners' appeal from the Insurance
Commissioner's decision for lack of merit. Hence, this petition.

Issue:
WON Philam didnt have the right to rescind the contract of insurance as
rescission must allegedly be done during the lifetime of the insured within
two years and prior to the commencement of action.

Held: No. Petition dismissed.

Ratio:
The Insurance Code states in Section 48:
Whenever a right to rescind a contract of insurance is given to the insurer
by any provision of this chapter, such right must be exercised previous to the
commencement of an action on the contract.
After a policy of life insurance made payable on the death of the insured
shall have been in force during the lifetime of the insured for a period of two
years from the date of its issue or of its last reinstatement, the insurer
cannot prove that the policy is void ab initio or is rescindable by reason of
the fraudulent concealment or misrepresentation of the insured or hisagent.

The so-called "incontestability clause" in the second paragraph prevents the


insurer from raising the defenses of false representations insofar as health
and previous diseases are concerned if the insurance has been in force for at
least two years during the insured's lifetime.
The policy was in force for a period of only one year and five months.
Considering that the insured died before the two-year period had lapsed,
respondent company is not, therefore, barred from proving that the policy is
void ab initio by reason of the insured's fraudulent concealment or
misrepresentation.
The "incontestability clause" added by the second paragraph of Section 48 is
in force for two years. After this, the defenses of concealment or
misrepresentation no longer lie.
The petitioners argue that no evidence was presented to show that the
medical terms were explained in a layman'slanguage to the insured. They
also argue that no evidence was presented by respondent company
to show that the questions appearing in Part II of the application for
insurance were asked, explained to and understood by the deceased so as to
prove concealment on his part. This couldnt be accepted because the
insured signed the form. He affirmed the correctness of all the entries.
The company records show that the deceased was examined by Dr.
Victoriano Lim and was found to be diabetic and hypertensive. He was also
found to have suffered from hepatoma. Because of the concealment made
by the deceased, the company was thus misled into accepting the risk and
approving his application as medically fit.t

MISREPRESENTATIONS
Form and when made
Sec. 36. A representation may be oral or written.
Sec. 37. A representation may be made at the time of, or before,
issuance of the policy.
Sec. 42. A representation must be presumed to refer to the date on
which the contract goes into effect.
Sec. 47. The provisions of this chapter apply as well to a
modification of a contract of insurance as to its original formation.
As to future
Sec. 39. A representation as to the future is to be deemed a
promise, unless it appears that it was merely a statement of belief
or expectation.
As to information
Sec. 43. When a person insured has no personal knowledge of a
fact, he may nevertheless repeat information which he has upon
the subject, and which he believes to be true, with the explanation
that he does so on the information of others; or he may submit the
information, in its whole extent, to the insurer; and in neither case
is he responsible for its truth, unless it proceeds from an agent of
the insured, whose duty it is to give the information.
Effects
Sec. 44. A representation is to be deemed false when the facts fail
to correspond with its assertions or stipulations.
Sec. 45. If a representation is false in a material point, whether
affirmative or promissory, the injured party is entitled to rescind
the contract from the time when the representation becomes
false. The right to rescind granted by this Code to the insurer is
waived by the acceptance of premium payments despite
knowledge of the ground for rescission. (As amended
by Batasang Pambansa Blg. 874).

As to age
Edillon v Manila Bankers Life G.R. No. L-34200 September 30, 1982
Facts:
Carmen O, Lapuz applied with Manila Bankers for insurance coverage against
accident and injuries. She gave the date of her birth as July 11, 1904. She
paid the sum of P20.00 representing the premium for which she was issued
the corresponding receipt. The policy was to be effective for 90 days.
During the effectivity, Carmen O. Lapuz died in a vehicular accident in the
North Diversion Road.
Petitioner Regina L. Edillon, a sister of the insured and the beneficiary in the
policy, filed her claim for the proceeds of the insurance. Her claim having
been denied, Regina L. Edillon instituted this action in the trial court.
The insurance corporation relies on a provision contained in the contract
excluding its liability to pay claims under the policy in behalf of "persons who
are under the age of sixteen (16) years of age or over the age of sixty (60)
years" They pointed out that the insured was over sixty (60) years of age
when she applied for the insurance coverage, hence the policy became void.
The trial court dismissed the complaint and ordered edillon to pay P1000.
The reason was that a policy of insurance being a contract of adhesion, it
was the duty of the insured to know the terms of the contract he or she is
entering into.
The insured could not have been qualified under the conditions stated in said
contract and should have asked for a refund of the premium.
Issue: Whether or not the acceptance by the insurance corporation of the
premium and the issuance of the corresponding certificate of insurance
should be deemed a waiver of the exclusionary condition of coverage stated
in the policy.
Held: Yes. Petition granted.
Ratio: The age of Lapuz was not concealed to the insurance company. Her
application clearly indicated her age of the time of filing the same to be
almost 65 years of age. Despite such information which could hardly be

overlooked, the insurance corporation received her payment of premium and


issued the corresponding certificate of insurance without question.
There was sufficient time for the private respondent to process the
application and to notice that the applicant was over 60 years of age and
cancel the policy.
Under the circumstances, the insurance corporation is already deemed in
estoppel. It inaction to revoke the policy despite a departure from the
exclusionary condition contained in the said policy constituted a waiver of
such condition, similar to Que Chee Gan vs. Law Union Insurance.
The insurance company was aware, even before the policies were issued,
that in the premises insured there were only two fire hydrants contrary to
the requirements of the warranty in question.
It is usually held that where the insurer, at the time of the issuance of a
policy of insurance, has knowledge of existing facts which, if insisted on,
would invalidate the contract from its very inception, such knowledge
constitutes a waiver of conditions in the contract inconsistent with the known
facts, and the insurer is stopped thereafter from asserting the breach of such
conditions.
To allow a company to accept one's money for a policy of insurance which it
then knows to be void and of no effect, though it knows as it must, that the
assured believes it to be valid and binding, is so contrary to the dictates of
honesty and fair dealing.
Capital Insurance & Surety Co., Inc. vs. - involved a violation of the provision
of the policy requiring the payment of premiums before the insurance shall
become effective. The company issued the policy upon the execution of a
promissory note for the payment of the premium. A check given subsequent
by the insured as partial payment of the premium was dishonored for lack of
funds. Despite such deviation from the terms of the policy, the insurer was
held liable.
... is that although one of conditions of an insurance policy is that "it shall
not be valid or binding until the first premium is paid", if it is silent as to the
mode of payment, promissory notes received by the company must be
deemed to have beenaccepted in payment of the premium. In other words, a
requirement for the payment of the first or initial premium inadvance or
actual cash may be waived by acceptance of a promissory note...
Test of Materiality
Sec. 46. The materiality of a representation is determined by the
same rules as the materiality of a concealment.

Pacific v CA G.R. No. L-41014 November 28, 1988

Facts:An open fire insurance policy, was issued to Paramount Shirt


Manufacturing by Oriental Assurance Corporation to indemnify P61,000.00,
caused by fire to the factorys stocks, materials and supplies.
The insured was a debtor of Pacific Banking in the amount of (P800,000.00)
and the goods described in the policy were held in trust by the insured for
Pacific Banking under trust receipts.
The policy was endorsed to Pacific Banking as mortgagee/ trustor of the
properties insured, with the knowledge and consent of private respondent to
the effect that "loss if any under this policy is payable to the Pacific Banking
Corporation".
A fire broke out on the premises destroying the goods contained in the
building.
The bank sent a letter of demand to Oriental for indemnity.
The company wasnt ready to give since it was awaiting the adjusters report.
The company then made an excuse that the insured had not filed any claim
with it, nor submitted proof of loss which is a clear violation of Policy
Condition No.11, as a result, determination of the liability of private
respondent could not be made.
Pacific Banking filed in the trial court an action for a sum of money for
P61,000.00 against Oriental Assurance.
At the trial, petitioner presented communications of the insurance adjuster to
Asian Surety revealing undeclared co-insurances with the following: P30,000
with Wellington Insurance; P25,000 with Empire Surety and P250,000 with
Asian Surety undertaken by insured Paramount on the same property
covered by its policy with Oriental whereas the only co-insurances declared

in the subject policy are those of P30,000.00 with Malayan P50,000.00 with
South Sea and P25.000.00 with Victory.
The defense of fraud, in the form of non-declaration of co-insurances which
was not pleaded in the answer, was also not pleaded in the Motion to
Dismiss.
The trial court denied the respondents motion. Oriental filed another motion
to include additional evidence of the co-insurance which could amount to
fraud.
The trial court still made Oriental liable for P 61,000. The CA reversed the
trial court decision. Pacific Banking filed a motion for reconsideration of the
said decision of the respondent Court of Appeals, but this was denied for lack
of merit.
Issues:
1. WON unrevealed co-insurances Violated policy conditions No. 3
2. WON the insured failed to file the required proof of loss prior to court
action.
Held: Yes. Petition dismissed.

Ratio:
1. Policy Condition No. 3 explicitly provides:
3. The Insured shall give notice to the Company of any insurance already
effected, or which may subsequently be effected, covering any of the
property hereby insured, and unless such notice be given and the particulars
of such insurance or insurances be stated in or endorsed on this Policy by or
on behalf of the Company before the occurrence of any loss or damage, all
benefit under this policy shall be forfeited.
The insured failed to reveal before the loss three other insurances. Had the
insurer known that there were many co-insurances, it could have hesitated
or plainly desisted from entering into such contract. Hence, the insured was
guilty of clear fraud.
Concrete evidence of fraud or false declaration by the insured was furnished
by the petitioner itself when the facts alleged in the policy under clauses
"Co-Insurances Declared" and "Other Insurance Clause" are materially
different from the actual number of co-insurances taken over the subject
property.

As the insurance policy against fire expressly required that notice should be
given by the insured of other insurance upon the same property, the total
absence of such notice nullifies the policy.
Petitioner points out that Condition No. 3 in the policy in relation to the
"other insurance clause" supposedly to have been violated, cannot certainly
defeat the right of the petitioner to recover the insurance as
mortgagee/assignee. Hence, they claimed that the purpose for which the
endorsement or assignment was made was to protect the
mortgagee/assignee against any untoward act or omission of the insured. It
would be absurd to hold that petitioner is barred from recovering the
insurance on account of the alleged violation committed by the insured.
It is obvious that petitioner has missed all together the import of subject
mortgage clause which specifically provides:
Loss, if any, under this policy, shall be payable to the PACIFIC BANKING
CORPORATION Manila mortgagee/trustor as its interest may appear, it being
hereby understood and agreed that this insurance as to the interest of the
mortgagee/trustor only herein, shall not be invalidated by any act or neglect
except fraud or misrepresentation, or arsonof the mortgagor or
owner/trustee of the property insured; provided, that in case the mortgagor
or owner/ trustee neglects or refuses to pay any premium, the mortgagee/
trustor shall, on demand pay the same.
The paragraph clearly states the exceptions to the general rule that
insurance as to the interest of the mortgagee, cannot be invalidated; namely:
fraud, or misrepresentation or arson. Concealment of the aforecited coinsurances can easily be fraud, or in the very least, misrepresentation.
Undoubtedly, it is but fair and just that where the insured who is primarily
entitled to receive the proceeds of the policy has by its fraud and/or
misrepresentation, forfeited said right.
Petitioner further stressed that fraud which was not pleaded as a defense in
private respondent's answer or motion to dismiss, should be deemed to have
been waived. It will be noted that the fact of fraud was tried by express or at
least implied consent of the parties. Petitioner did not only object to the
introduction of evidence but on the contrary, presented the very evidence
that proved its existence.
2. Generally, the cause of action on the policy accrues when the loss occurs,
But when the policy provides that no action shall be brought unless the claim
is first presented extrajudicially in the manner provided in the policy, the
cause of action will accrue from the time the insurer finally rejects the claim
for payment

In the case at bar, policy condition No. 11 specifically provides that the
insured shall on the happening of any loss or damage give notice to the
company and shall within fifteen (15) days after such loss or damage deliver
to the private respondent (a) a claim in writing giving particular account as
to the articles or goods destroyed and the amount of the loss or damage and
(b) particulars of all other insurances, if any.
Twenty-four days after the fire did petitioner merely wrote letters to private
respondent to serve as a notice of loss. It didnt even furnish other
documents. Instead, petitioner shifted upon private respondent the burden of
fishing out the necessary information to ascertain the particular account of
the articles destroyed by fire as well as the amount of loss. Since the
required claim by insured, together with the preliminary submittal of relevant
documents had not been complied with, it follows that private respondent
could not be deemed to have finally rejected petitioner's claim and therefore
there was no cause of action.
It appearing that insured has violated or failed to perform the conditions
under No. 3 and 11 of the contract, and such violation or want of
performance has not been waived by the insurer, the insured cannot recover,
much less the herein petitioner.

Title: Eguaras v. Great Eastern Life Ass. Co.


Topic: Misrepresentation, effect (Section 45)
Facts: On April 14, 1913, counsel for Francisca Eguaras filed a written
complaint in the said Laguna court, alleging as a cause of action that about
October 14, 1912, her son-in-law Dominador Albay had applied in writing to
the defendant insurance company to insure his life for the sum of P5,000,
naming as the beneficiary in case of his death the plaintiff Francisca Eguaras;
that after compliance with the requisites and the investigation carried on by
the defendant company, and it had been satisfied concerning the physical
condition of the applicant, it accepted the application for insurance and on
November 6, 1912, issued policy No. 5592, Exhibit A, which has been made a
part of the complaint, whereby the said insurance company insured the life
of the said Dominador Albay in the sum of P5,000, payable in the event of his
death to Francisca Eguaras; that on December 6, 1912, said policy No. 5592
being in force, the insured Dominador Albay, died in the municipality of
Santa Cruz, Laguna, and despite the fact that the beneficiary submitted
satisfactory proofs of his death and that the defendant company investigated
the event, still it refused and continues to refuse to pay to the plaintiff the
value of the policy, Exhibit A, thereby causing damages estimated at P1,000.

The court was therefore asked to render judgment against the Great Eastern
Life Assurance Company, Ltd., and its general agent, West G. Smith, by
sentencing them to pay to the plaintiff the sum of P5,000, the value of policy
No. 5592, plus the sum of P1,000 for damages inflicted upon them, in
addition to the costs of the suit.
The demurrer filed to the foregoing complaint having been overruled,
counsel for the insurance company and for West G. Smith replied thereto,
admitting the allegations of the complaint with respect to the legal status of
the parties by denying all the rest, and setting forth in special defense that
the insurance policy issued in the name of Dominador [Albay] had been
obtained through fraud and deceit known and consented to by the interested
parties and is therefore completely illegal, void, and ineffective; wherefore he
prayed that the defendants be absolved from the complaint, with the costs
against the plaintiff.

Issue:WON the life insurance obtained is legal and valid or whether on the
contrary it was issued through fraud and deceit, and in such case, whether
the defendant, The Great Eastern Life Assurance Company, Ltd., is still under
obligation to pay the value thereof to the plaintiff.

Ruling: It appears from the record that the insured had knowledge of the
false replied contained in the two applications for insurance and knowing
permitted fraud to be practised upon the insurance company, for in his
acknowledgment and consent his mother-in-law was designated as the
beneficiary of the insurance, despite the fact that he had children and his
mother was still living. In the present case the fraud consisted in the fact
that a healthy and robust person was substituted in place of insured invalid
when Dr. Vidal made the physical examination of the one who seeking to be
insured, for the real person who desired to be insured and who ought to have
been examined was in bad health on and before the date of executing the
insurance contract of which facts the insured Dominador Albay and the
insurance agent Ponciano Remigio had full knowledge.
It is therefore proven that the signatures on the insurance applications
reading "Dominador Albay" are false and forged; that the person who
presented himself to Dr. Vidal to be examined was not the real Dominador
Albay, but another different person; that at the time of the application for
insurance and the issuance of the policy which is the subject matter of this
suit the real Dominador Albay was informed of all those machinations,

wherefore it is plain that the insurance contract between the defendant and
Dominador Albay is null and void because it is false, fraudulent and illegal.
Article 1269 of the Civil Code states:
There is deceit when by words or insidious machinations on the
part of one of the contracting parties the other is induced to execute a
contract which without them he would not have made.
It is essential to the nature of the deceit, to which the foregoing article
refers, that said deceit be prior to or contemporaneous with the consent that
is a necessary requisite for perfecting the contract, but not that it may have
occurred or happened thereafter. A contract is therefore deceitful, for the
execution whereof the consent of one of the parties has been secured by
means of fraud, because he was persuaded by words or insidious
machinations, statements or false promises, and a defective consent wrung
from him, even though such do not constitute estafa or any other criminal
subject to the penal law.
With this array of circumstantial evidence derived from facts duly
proven as a result of the present suit, we get, if not a moral certainly, at least
a full conviction that when Castor Garcia presented himself to be examined
by the physician Vidal in place of Dominador Albay, serious deceit occurred
in perfecting the insurance contract, for had the agent of the company not
been deceived it would not have granted the insurance applied for by Albay,
nor would it have executed the contract by virtue of whereof payment is
claimed of the value of policy obtained through fraud; and consequently on
such assumptions it is improper, nor is it permitted by the law, to order
collection of the amount claimed.
In a contract executed with the requisites fixed in article 1261, one of the
contracting parties may have given his consent through error, violence,
intimidation, or deceit, and in any of such cases the contract is void, even
though, despite this nullity, no crime was committed. (Article 1265, Civil
Code.) There may not have been estafa in the case at bar, but it was
conclusively demonstrated by the trial that deceit entered into the insurance
contract, fulfillment whereof is claimed, and therefore the conclusions
reached by the court in the judgment it rendered in the criminal proceedings
for estafa do not affect this suit, nor do they influence the decision proper
herein, nor can they produce in the present suit, over the exception of the
defendant, the force of res adjudicata.
WARRANTIES

Defined: A warranty in an insurance policy is a promise by


the insured party that statements affecting the validity of the contract are
true. Mostinsurance contracts require the insured to make
certain warranties. ... Only misrepresentations on conditions
and warranties in the contract give an insurer such rights.
KINDS
Sec. 67. A warranty is either expressed or implied.
Sec. 68. A warranty may relate to the past, the present, the future,
or to any or all of these.
Sec. 71. A statement in a policy of matter relating to the person or
thing insured, or to the risk, as a fact, is an express warranty
thereof.
Sec. 72. A statement in a policy which imparts that it is intended to
do or not to do a thing which materially affects the risk, is a
warranty that such act or omission shall take place.

Qua v Law Union. G.R. No. L-4611 December 17, 1955

Facts:Qua owned 4 warehouses used for the storage of copra and hemp.
They were insured with the Law Union.
Fire broke out and completely destroyed 3 bodegas. The plaintiff submitted
claims totalling P398,562.81. The Insurance Company resisted payment on
the grounds that the fire had been deliberately caused by the insured or by
other persons in connivance with him.
Que Chee Gan and his brother were tried for arson, but were acquitted by
the trial court. As regards the insurance claim, the trial court ruled in favor
of Qua and entitled him to recover more than Php 300,000 for indemnities
from the insurance company. Hence, the company appealed to the SC.
In its first assignment of error, the insurance company alleged that the trial
Court should have held that the policies were avoided for breach of warranty.
The contract noted that fire hydrants were required in a particular
measurement of space (every 150 feet). Hence, they argued that since the
bodegas insured had an external wall perimeter of 500 meters, the appellee
should have 11 fire hydrants in the compound, and that he actually had only
2, with a further pair.
Issues:

1. WON the insurance company can void the policies it had issued
2. WON the insured violated the "Hemp Warranty" provisions of the policy
against the storage of gasoline
3. WON the insured planned the destruction of the bodega

Held: No. No. No.


Ratio:
1. The insurer, who at the time of issuance, has knowledge of existing facts
which would invalidate the contract from the beginning, such constitutes a
waiver of conditions in the contract inconsistent with the facts, and the
insurer is stopped thereafter from asserting the breach of such conditions.
Also, an insurance company intends to executed a valid contract in return for
the premium received; and when the policy contains a condition which
renders it voidable at its inception, and this result is known to the insurer, it
will be presumed to have intended to waive the conditions and to execute a
binding contract, rather than to have deceived the insured into thinking he is
insured when in fact he is not.
The appellant is barred estoppel to claim violation of the so-called fire
hydrants warranty, because it knew the number of hydrants demanded
therein never existed from the very beginning and issued the policies.
To allow a company to accept one's money for a policy of insurance which it
then knows to be void and of no effect, though it knows as it must, that the
assured believes it to be valid and binding, is so contrary to the dictates of
honesty and fair dealing, and so closely related to positive fraud, as to
the abhorrent to fair-minded men.
The appellant company so worded the policies that while exacting the
greater number of fire hydrants and appliances, it kept the premium discount
at the minimum of 2 1/2%, thereby giving the insurance company a double
benefit. Suchabnormal treatment of the insured strongly points at an abuse
of the insurance company's selection of the words and terms of the contract,
over which it had absolute control.
Receipt of Premiums or Assessments after Cause for Forfeiture Other than
Nonpayment. It is a well settled rule of law that an insurer which with
knowledge of facts entitling it to treat a policy as no longer in force, receives
and accepts a premium on the policy, estopped to take advantage of the
forfeiture. It cannot treat the policy as void for the purpose of defense to an

action to recover for a loss thereafter occurring and at the same time treat it
as valid for the purpose of earning and collecting further premiums.
Moreover, taking into account the well known rule that ambiguities or
obscurities must be strictly interpreted against the party that caused them,
the "memo of warranty" invoked by appellant bars the latter from
questioning the existence of the appliances called for in the insured premises
2. The ambiguity must be held strictly against the insurer and liberally in
favor of the insured, specially to avoid a forfeiture. So long as insurance
companies insist upon the use of ambiguous, intricate and technical
provisions, which conceal rather than frankly disclose, their own intentions,
the courts must, in fairness to those who purchase insurance, construe
everyambiguity in favor of the insured.
Appellee admitted that there were 36 cans of gasoline in the building
designed. It However, gasoline is not specifically mentioned among the
prohibited articles listed in the so-called "hemp warranty." The cause relied
upon by the insurer speaks of "oils", and is uncertain because, "Oils" usually
mean "lubricants" and not gasoline or kerosene.
If the company intended to rely upon a condition of that character, it ought
to have been plainly expressed in the policy.
The contract of insurance is one of perfect good faith not for the
insured alone, but equally so for the insurer; in fact, it is mere so for the
latter, since its dominant bargaining position carries with it stricter
responsibility.
Also, the gasoline kept in Bodega No. 2 was only incidental to his business,
being no more than a customary 2 day's supply for the five or six motor
vehicles used for transporting of the stored merchandise. "It is well settled
that the keeping of inflammable oils on the premises though prohibited by
the policy does not void it if such keeping is incidental to thebusiness."
3. It was unlikely that Qua burned the warehouse to defraud the company
because he had the resources to pay off the National Bank in a short time.
Also, no motive appears for attempt to defraud the insurer. While the
acquittal of the insured in the arson case is not res judicata on the present
civil action, the insurer's evidence, to judge from the decision in the criminal
case, is practically identical in both cases and must lead to the same result,
since the proof to establish the defense of connivance at the fire in order to
defraud the insurer "cannot be materially less convincing than that required
in order to convict the insured of the crime of arson."

As to the defense that the burned bodegas could not possibly have contained
the quantities of copra and hemp stated in the fire claims, the insurer relied
on its adjuster investigator who examined the premises during and after the
fire. His testimony, however, was based on inferences from the photographs
and traces found after the fire, and must yield to the contradictory testimony
of those who actually saw the contents of the bodegas shortly before the fire,
while inspecting them for the mortgagee Bank.
Young vs. Midland Textile insurance company
Facts: The purpose of the present action is to recover the sum of P3,000
upon an insurance policy. The lower court rendered a judgment in favor of
the plaintiff and against the defendant for the sum of P2,708.78, and costs.
From that judgment the defendant appealed to this court.
The undisputed facts upon which said action is based are as follows:
The plaintiff occupied a building at '321 Calle Claveria, as a residence
and bodega (storehouse). On the 29th of May, 1912, the defendant, in
consideration of the payment of a premium of P60, entered into a contract of
insurance with the plaintiff promising to pay to the plaintiff the sum of
P3,000, in case said residence and bodega and contents should be destroyed
by fire. One of the conditions of said contract was that no hazardous goods
be stored or kept in the building.
On the 4th or 5th of February, 1913, the plaintiff placed in said
residence and bodega three boxes which belonged to him and which were
filled with fireworks for the celebration of the Chinese new year.
On the 18th day of March, 1913, said residence and bodega and the
contents thereof were partially destroyed. Fireworks were found in a part of
the building not destroyed by the fire; that they in no way contributed to the
fire, or to the loss occasioned thereby.
Issue: Whether or not the placing of said fireworks in the building insured,
under the conditions above enumerated, they being "hazardous goods," is a
violation of the terms of the contract of insurance.
Held: Yes.
The word "stored" has been defined to be a deposit in a store or
warehouse for preservation or safe keeping; to put away for future use,
especially for future consumption; to place in a warehouse or other place of
deposit for safe keeping. Said definition does not include a deposit in a store,
in small quantities, for daily use. "Daily use" precludes the idea of deposit for
preservation or safe keeping, as well as a deposit for future consumption or
safe keeping.

A violation of the terms of a contract of insurance, by either party, will


constitute the basis for a termination of the contractual relations, at the
election of the other. The right to terminate the contractual relations exists
even though the violation was not the direct cause of the loss. In the present
case, the deposit of the "hazardous goods," in the building insured, was a
violation of the terms of the contract. Although the hazardous goods did not
contribute to the loss, the insurer, at his election, was relieved from liability
Said deposit created a new risk, not included in the terms of the contract.
The insurer had neither been paid, nor had he entered into a contract, to
cover the increased risk.
Contracts of insurance are contracts of indemnity, upon the terms and
conditions specified therein. Parties have a right to impose such reasonable
conditions at the time of the making of the contract as they deem wise and
necessary. The rate of premium is measured by the character of the risk
assumed. The insurer, for a comparatively small consideration, undertakes to
guarantee the insured against loss or damage, upon the terms and
conditions agreed upon, and upon no other. When the insurer is called upon
to pay, in case of loss, he may justly insist upon a fulfillment of the terms of
the contract. If the insured cannot bring himself within the terms and
conditions of the contract, he is not entitled to recover for any loss suffered.
The terms of the contract constitute the measure of the insurer's liability. If
the contract has been terminated, by a violation of its terms on the part of
the insured, there can be no recovery. Compliance with the terms of the
contract is a condition precedent to the right of recovery. Courts cannot
make contracts for the parties. While contracts of insurance are construed
most favorably to the insured yet they must be construed according to the
sense and meaning of the terms which the parties themselves have used.
Astute and subtle distinctions should not be permitted, when the language of
the contract is plain and unambiguous. Such distinctions tend to bring the
law itself into disrepute.
The judgment of the lower court is revoked and the defendant is
relieved from any responsibility under said complaint, and, without any
finding as to costs.

Form
Sec. 69. No particular form of words is necessary to create a
warranty.

When Omission does not avoid


Sec. 73. When, before the time arrives for the performance of a
warranty relating to the future, a loss insured against happens, or
performance becomes unlawful at the place of the contract, or
impossible, the omission to fulfill the warranty does not avoid the
policy.
Effect of Breach
Sec. 74. The violation of a material warranty, or other material
provision of a policy, on the part of either party thereto, entitles
the other to rescind.
Sec. 75. A policy may declare that a violation of specified
provisions thereof shall avoid it, otherwise the breach of an
immaterial provision does not avoid the policy.
Sec. 76. A breach of warranty without fraud merely exonerates an
insurer from the time that it occurs, or where it is broken in its
inception, prevents the policy from attaching to the risk.

Pioneer v Yap G.R. No. L-36232 December 19, 1974

Facts: Respondent Oliva Yap was the owner of a store in a two-storey building
where she sold shopping bags and footwear. Chua Soon Poon, her son-in-law,
was in charge of the store.
Yap took out a Fire Insurance Policy No. 4216 from Pioneer Insurance with a
value of P25,000.00 covering her stocks, office furniture, fixtures and fittings.
Among the conditions in the policy executed by the parties are the following:
unless such notice be given and the particulars of such insurance or
insurances be stated in, or endorsed on this Policy by or on behalf of the
Company before the occurrence of any loss or damage, all benefits under
this Policy shall be forfeited Any false declaration or breach or this
condition will render this policy null and void.
Another insurance policy for P20,000.00 issued by Great American covering
the same properties. The endorsement recognized co-insurance by
Northwest for the same value.
Oliva Yap took out another fire insurance policy for P20,000.00 covering the
same properties from the Federal Insurance Company, Inc., which was
procured without notice to and the written consent of Pioneer.

A fire broke out in the building, and the store was burned. Yap filed an
insurance claim, but the same was denied for a breach.
Oliva Yap filed a case for payment of the face value of her fire insurance
policy. The insurance company refused to pay because she never informed
Pioneer of another insurer. The trial court decided in favor of Yap. The CA
affirmed.

Issue: Whether or not petitioner should be absolved from liability on the


Pioneeer policy on account of any violation of the co-insurance clause

Held: No. Petition dismissed.


Ratio: There was a violation. The insurance policy for P20,000.00 issued by
the Great American, ceased to be recognized by them as a co-insurance
policy.
The endorsement shows the clear intention of the parties to recognize on the
date the endorsement was made, the existence of only one co-insurance, the
Northwest one. The finding of the Court of Appeals that the Great American
Insurance policy was substituted by the Federal Insurance policy is indeed
contrary to said stipulation.
Other insurance without the consent of Pioneer would avoid the contract. It
required no affirmative act of election on the part of the company to make
operative the clause avoiding the contract, wherever the specified conditions
should occur. Its obligations ceased, unless, being informed of the fact, it
consented to the additional insurance.
The validity of a clause in a fire insurance policy to the effect that the
procurement of additional insurance without the consent of the insurer
renders the policy void is in American jurisprudence.
Milwaukee Mechanids' Lumber Co., vs. Gibson- "The rule in this state and
practically all of the states is to the effect that a clause in a policy to the
effect that the procurement of additional insurance without the consent of
the insurer renders the policy void is a valid provision.
In this jurisdiction, General Insurance & Surety Corporation vs. Ng HuaThe annotation then, must be deemed to be a warranty that the property
was not insured by any other policy. Violation thereof entitled the insurer to
rescind. Furthermore, even if the annotations were overlooked the defendant
insurer would still be free from liability because there is no question that the
policy issued by General Indemnity has not been stated in nor endorsed on

Policy No. 471 of defendant. The obvious purpose of the aforesaid


requirement in the policy is to prevent over-insurance and thus avert the
perpetration of fraud where a fire would be profitable to the insured.

New Life v CA G.R. No. 94071 March 31, 1992

Facts: Julian Sy, owner of New Life, insured his building in 3


different insurance agencies for 350,000, 1,000,000, and 200,000. When his
building and the goods inside burned down, he claimed for insurance
indemnities, but these were rejected by the three companies for violation of
policy conditions.
Sy filed for 3 different suits in the trial court, where he won all suits against
the insurance companies. The court of appealsreversed the decision of the
trial court.
Issue: Did the petitioner violate conditions 3 and 27 of the three insurance
policies, thereby foreiting collection of indemnities?
Held: Yes.
Ratio: Condition 3. The insured shall give notice to the Company of any
insurance or insurances already effected, or which may subsequently be
effected, covering any of the property or properties consisting of stocks in
trade, goods in process and/or inventories only hereby insured, and unless
such notice be given and the particulars of such insurance or
insurances be stated therein or endorsed on this policy pursuant to
Section 50 of the Insurance Code, by or on behalf of the Company before the
occurrence of any loss or damage, all benefits under this policy shall be
deemed forfeited, provided however, that this condition shall not apply when
the total insurance or insurances in force at the time of loss or damage not
more than P200,000.00.
Sy never disclosed co-insurance in the contracts he entered into with the
three corporations. The insured is specifically required to disclose the
insurance that he had contracted with other companies. Sy also contended
that the insuranceagents knew of the co-insurance. However, the theory of
imputed knowledge, that the knowledge of the agent is presumed to be
known by the principal, is not enough.
When the words of the document are readily understandable by an ordinary
reader, there is no need for constructionanymore.

The conformity of the insured to the terms of the policy is implied with his
failure to disagree with the terms of the contract.
Since Sy, was a businessman, it was incumbent upon him to read the
contracts.
Pioneer Insurance and Surety Corporation vs. Yap- The obvious purpose of
the aforesaid requirement in the policy is to prevent over-insurance and thus
avert the perpetration of fraud. The public, as well as the insurer, is
interested in preventing the situation in which a fire would be profitable to
the insured.
Also, policy condition 15 was used. It stated: 15.. . . if any false declaration
be made or used in support thereof, . . . all benefits under this Policy shall be
forfeited . . .
As for condition number 27, the stipulation read:
27. Action or suit clause. If a claim be made and rejected and an action or
suit be not commenced either in the Insurance Commission or any court of
competent jurisdiction of notice of such rejection, or in case
of arbitration taking place as provided herein, within twelve (12) months
after due notice of the award made by the arbitrator or arbitrators or umpire,
then the claim shall for all purposes be deemed to have been abandoned and
shall not thereafter be recoverable hereunder.
This is regarding Sys claim for one of the companies. Recovery was filed in
court by petitioners only on January 31, 1984, or after more than one (1)
year had elapsed from petitioners' receipt of the insurers' letter of denial on
November 29, 1982. This made it void.
MEANING OF WARRANTY CONDITION
PRUDENTIAL GUARANTEE and ASSURANCE INC., vs. TRANS-ASIA
SHIPPING
Facts: TRANS-ASIA is the owner of the vessel M/V Asia Korea. In consideration
of payment of premiums, PRUDENTIAL insured M/V Asia Korea for
loss/damage of the hull and machinery arising from perils, inter alia, of fire
and explosion for the sum of P40 Million, beginning from the period of July 1,
1993 up to July 1, 1994.
On October 25, 1993, while the policy was in force, a fire broke out while
[M/V Asia Korea was] undergoing repairs at the port of Cebu. On October 26,
1993 TRANS-ASIA filed its notice of claim for damage sustained by the vessel

evidenced by a letter/formal claim. TRANS-ASIA reserved its right to


subsequently notify PRUDENTIAL as to the full amount of the claim upon final
survey and determination by average adjuster Richard Hogg International
(Phil.) of the damage sustained by reason of fire.
TRANS-ASIA executed a document denominated "Loan and Trust receipt", a
portion of which states that Received from Prudential Guarantee and
Assurance, Inc., the sum of PESOS THREE MILLION ONLY (P3,000,000.00) as a
loan without interest under Policy No. MH 93/1353 [sic], repayable only in the
event and to the extent that any net recovery is made by Trans-Asia Shipping
Corporation, from any person or persons, corporation or corporations, or
other parties, on account of loss by any casualty for which they may be liable
occasioned by the 25 October 1993: Fire on Board."
PRUDENTIAL later on denied Trans-Asias claim in stated in a letter that "After
a careful review and evaluation of your claim arising from the abovecaptioned incident, it has been ascertained that you are in breach of policy
conditions, among them "WARRANTED VESSEL CLASSED AND CLASS
MAINTAINED". Accordingly, we regret to advise that your claim is not
compensable and hereby DENIED." and asked for the return of the
3,000,000.
TRANS-ASIA filed a Complaint for Sum of Money against PRUDENTIAL with
the RTC of Cebu City, wherein TRANS-ASIA sought
the
amount
of
P8,395,072.26 from PRUDENTIAL, alleging that the same represents
the balance of the indemnity due upon the insurance policy in the total
amount of P11,395,072.26. TRANS-ASIA similarly sought interest at 42% per
annum citing Section 243 of Presidential Decree No. 1460, otherwise known
as the "Insurance Code," asamended.
PRUDENTIAL denied the material allegations of the Complaint and interposed
the defense that TRANS-ASIA breached insurance policy conditions, in
particular: PRUDENTIAL posits that TRANS-ASIA violated an express
and
material warranty in the subject insurance contract, i.e., Marine
Insurance Policy No. MH93/1363, specifically Warranty Clause No. 5 thereof,
which stipulates that the insured vessel, "M/V ASIA KOREA" is required to be
CLASSED AND CLASS MAINTAINED. According to PRUDENTIAL, on 25 October
1993, or at the time of the occurrence of the fire, "M/V ASIA KOREA" was in
violation of the warranty as it was not CLASSED AND CLASS MAINTAINED.
PRUDENTIAL submits that Warranty Clause No. 5 was a condition precedent
to the recovery of TRANS-ASIA under the policy, the violation of which
entitled PRUDENTIAL to rescind the contract under Sec. 74 of the Insurance
Code.
By way of a counterclaim, PRUDENTIAL sought a refund of
P3,000,000.00, which it allegedly advanced to TRANS-ASIA by way of a loan

without interest and without prejudice to the final evaluation of the claim,
including the amounts of P500,000.00, for survey fees and P200,000.00,
representing attorneys fees.
Trial court ruled in favor of Prudential. It ruled that a determination of the
parties liabilities hinged on whether TRANS- ASIA violated and breached
the policy conditions
on WARRANTED VESSEL CLASSED AND CLASS
MAINTAINED. It interpreted the provision to mean that TRANS-ASIA is
required to maintain the vessel at a certain class at all times pertinent during
the life of the policy. According to the court a quo, TRANS-ASIA failed to prove
compliance of the terms of the warranty, the violation thereof entitled
PRUDENTIAL to rescind the contract.
The court of appeals reversed the decision. It ruled that PRUDENTIAL, as the
party asserting the non-compensability of the loss had the burden of proof to
show that TRANS-ASIA breached the warranty, which burden it failed to
discharge. PRUDENTIAL cannot rely on the lack of certification to
the effect that TRANS-ASIA was CLASSED AND CLASS MAINTAINED
as its sole basis for reaching the conclusion that the warranty was breached.
It opined that the lack of a certification does not necessarily mean that
the warranty was breached by TRANS-ASIA. Instead, it considered
PRUDENTIALs admission that at the time the insurance contract was entered
into between the parties, the vessel was properly classed by Bureau Veritas,
a classification society recognized by the industry. It similarly gave weight to
the fact
that
it
was
the
responsibility
of
Richards
Hogg
International
(Phils.)
Inc.,
the
average
adjuster
hired
by
PRUDENTIAL, to secure a copy of such certification to support its conclusion
that mere absence of a certification does not warrant denial of TRANS-ASIAs
claim under the insurance policy.
Issue: WON Trans-Asia breached the warranty stated in the insurance policy,
thus absolving Prudential from paying Trans-Asia.
Ruling: No.
Rationale:
As found by the Court of Appeals and as supported by the records, Bureau
Veritas is a classification society recognized in the marine industry. As it is
undisputed that TRANS-ASIA was properly classed at the time the contract of
insurance was entered into, thus, it becomes incumbent upon PRUDENTIAL
to show evidence that the status of TRANS-ASIA as being properly CLASSED
by Bureau Veritas had shifted in violation of the warranty. Unfortunately,
PRUDENTIAL failed to support the allegation.

The lack of a certification in PRUDENTIALs records to the effect that TRANSASIAs "M/V Asia Korea" was CLASSED AND CLASS MAINTAINED at the time of
the occurrence of the fire cannot be tantamount to the conclusion that
TRANS-ASIA in fact breached the warranty contained in the policy.
It was likewise the responsibility of the average adjuster, Richards Hogg
International (Phils.), Inc., to secure a copy of such certification, and the
alleged breach of TRANS-ASIA cannot be gleaned from the average adjusters
survey report, or adjustment of particular average per "M/V Asia Korea" of
the 25 October 1993 fire on board.
The Supreme Court is not unmindful of the clear language of Sec. 74 of the
Insurance Code which provides that, "the violation of a material warranty or
other material provision of a policy on the part of either party thereto,
entitles the other to rescind." It is generally accepted that "a warranty is a
statement or promise set forth in the policy, or by reference incorporated
therein, the untruth or non-fulfillment of which in any respect, and without
reference to whether the insurer was in fact prejudiced by such untruth or
non- fulfillment, renders the policy voidable by the insurer."
However, it is similarly indubitable that for the breach of a warranty to avoid
a policy, the same must be duly shown by the party alleging the same.
We cannot sustain an allegation that is unfounded. Consequently,
PRUDENTIAL, not having shown that TRANS-ASIA breached the warranty
condition, CLASSED AND CLASS MAINTAINED, it remains that TRANS-ASIA
must be allowed to recover its rightful claims on the policy.
Assuming arguendo that TRANS-ASIA violated the policy condition on
WARRANTED VESSEL CLASSED AND CLASS MAINTAINED, PRUDENTIAL made
a valid waiver of the same.
PRUDENTIAL can be deemed to have made a valid waiver of TRANS-ASIAs
breach of warranty as alleged. Because after the loss, Prudential renewed
the insurance policy of Trans-Asia for two (2) consecutive years, from noon of
01 July 1994 to noon of 01 July 1995, and then again until noon of 01 July
1996. This renewal is deemed a waiver of any breach of warranty.
PRUDENTIAL, in renewing TRANS-ASIAs insurance policy for two consecutive
years after the loss covered by Policy No. MH93/1363, was considered to
have waived TRANS-ASIAs breach of the subject warranty, if any. Breach of a
warranty or of a condition renders the contract defeasible at the option of
the insurer; but if he so elects, he may waive his privilege and power to
rescind by the mere expression of an intention so to do. In that event his
liability under the policy continues as before. There can be no clearer
intention of the waiver of the alleged breach than the renewal of the policy

insurance granted by PRUDENTIAL to TRANS-ASIA in MH94/1595 and


MH95/1788, issued in the years 1994 and 1995, respectively.

Warranties in fire insurance policies


Sec. 168. An alteration in the use or condition of a thing insured
from that to which it is limited by the policy made without the
consent of the insurer, by means within the control of the insured,
and increasing the risks, entitles an insurer to rescind a contract
of fire insurance.
Sec. 169. An alteration in the use or condition of a thing insured
from that to which it is limited by the policy, which does not
increase the risk, does not affect a contract of fire insurance.
Sec. 170. A contract of fire insurance is not affected by any act of
the insured subsequent to the execution of the policy, which does
not violate its provisions, even though it increases the risk and is
the cause of the loss.
Title: Young vs. Midland Textile insurance company
Facts: The purpose of the present action is to recover the sum of P3,000
upon an insurance policy. The lower court rendered a judgment in favor of
the plaintiff and against the defendant for the sum of P2,708.78, and costs.
From that judgment the defendant appealed to this court.
The undisputed facts upon which said action is based are as follows:
The plaintiff occupied a building at '321 Calle Claveria, as a residence
and bodega (storehouse). On the 29th of May, 1912, the defendant, in
consideration of the payment of a premium of P60, entered into a contract of
insurance with the plaintiff promising to pay to the plaintiff the sum of
P3,000, in case said residence and bodega and contents should be destroyed
by fire. One of the conditions of said contract was that no hazardous goods
be stored or kept in the building.

On the 4th or 5th of February, 1913, the plaintiff placed in said


residence and bodega three boxes which belonged to him and which were
filled with fireworks for the celebration of the Chinese new year.
On the 18th day of March, 1913, said residence and bodega and the
contents thereof were partially destroyed. Fireworks were found in a part of
the building not destroyed by the fire; that they in no way contributed to the
fire, or to the loss occasioned thereby.
Issue: Whether or not the placing of said fireworks in the building insured,
under the conditions above enumerated, they being "hazardous goods," is a
violation of the terms of the contract of insurance.
Held: Yes.
The word "stored" has been defined to be a deposit in a store or
warehouse for preservation or safe keeping; to put away for future use,
especially for future consumption; to place in a warehouse or other place of
deposit for safe keeping. Said definition does not include a deposit in a store,
in small quantities, for daily use. "Daily use" precludes the idea of deposit for
preservation or safe keeping, as well as a deposit for future consumption or
safe keeping.
A violation of the terms of a contract of insurance, by either party, will
constitute the basis for a termination of the contractual relations, at the
election of the other. The right to terminate the contractual relations exists
even though the violation was not the direct cause of the loss. In the present
case, the deposit of the "hazardous goods," in the building insured, was a
violation of the terms of the contract. Although the hazardous goods did not
contribute to the loss, the insurer, at his election, was relieved from liability
Said deposit created a new risk, not included in the terms of the contract.
The insurer had neither been paid, nor had he entered into a contract, to
cover the increased risk.
Contracts of insurance are contracts of indemnity, upon the terms and
conditions specified therein. Parties have a right to impose such reasonable
conditions at the time of the making of the contract as they deem wise and
necessary. The rate of premium is measured by the character of the risk
assumed. The insurer, for a comparatively small consideration, undertakes to
guarantee the insured against loss or damage, upon the terms and
conditions agreed upon, and upon no other. When the insurer is called upon
to pay, in case of loss, he may justly insist upon a fulfillment of the terms of
the contract. If the insured cannot bring himself within the terms and
conditions of the contract, he is not entitled to recover for any loss suffered.
The terms of the contract constitute the measure of the insurer's liability. If

the contract has been terminated, by a violation of its terms on the part of
the insured, there can be no recovery. Compliance with the terms of the
contract is a condition precedent to the right of recovery. Courts cannot
make contracts for the parties. While contracts of insurance are construed
most favorably to the insured yet they must be construed according to the
sense and meaning of the terms which the parties themselves have used.
Astute and subtle distinctions should not be permitted, when the language of
the contract is plain and unambiguous. Such distinctions tend to bring the
law itself into disrepute.
The judgment of the lower court is revoked and the defendant is
relieved from any responsibility under said complaint, and, without any
finding as to costs.

S-ar putea să vă placă și